Đến nội dung

Hình ảnh

$\frac{xy}{1+z^2}+\frac{yz}{1+x^2}-y^3(\frac{1}{x^3}+\frac{1}{z^3})\leq \frac{-3}{2}$


  • Please log in to reply
Chủ đề này có 1 trả lời

#1
Thao Meo

Thao Meo

    Trung sĩ

  • Thành viên
  • 122 Bài viết

Cho x,y,z>0 thỏa mãn x+z$\leq$ 2y và $x^2+y^2+z^2=1$ . Chứng minh

$\frac{xy}{1+z^2}+\frac{yz}{1+x^2}-y^3(\frac{1}{x^3}+\frac{1}{z^3})\leq \frac{-3}{2}$


:icon11:  Trong cuộc sống không có gì là đẳng thức , tất cả đều là bất đẳng thức :ukliam2:  :ukliam2: 


#2
quoccuonglqd

quoccuonglqd

    Thượng sĩ

  • Thành viên
  • 219 Bài viết
Áp dụng AM-GM,C-S:$\frac{xy}{1+z^{2}}+\frac{yz}{1+x^{2}}=\frac{xy}{x^{2}+y^{2}+2z^{2}}+\frac{yz}{y^{2}+z^{2}+2x^{2}}\leqslant \frac{x^{2}}{4(x^{2}+z^{2})}+\frac{z^{2}}{4(x^{2}+z^{2})}+\frac{y^{2}}{4}(\frac{1}{y^{2}+x^{2}}+\frac{1}{y^{2}+z^{2}})\leqslant \frac{1}{4}+\frac{y^{2}}{16y^{2}}+\frac{y^{2}}{16y^{2}}+\frac{y^{2}}{16}(\frac{1}{z^{2}}+\frac{1}{x^{2}})=\frac{15}{8}+\frac{y^{2}}{16}(\frac{1}{x^{2}}+\frac{1}{z^{2}})$
Lại có $\frac{15}{16}y^{3}(\frac{1}{x^{3}}+z^{3})\geqslant \frac{15}{8}$ (với $2y\geqslant x+z$)
Ta cần chứng minh $y^{3}(\frac{1}{x^{3}}+\frac{1}{z^{3}})\geqslant y^{2}(\frac{1}{x^{2}}+\frac{1}{z^{2}})\Leftrightarrow x^{4}+y^{4}+x^{3}z+xz^{3}\geqslant 2x^{3}z+2xz^{3}$(luôn đúng)





0 người đang xem chủ đề

0 thành viên, 0 khách, 0 thành viên ẩn danh